3
$\begingroup$

Let $(f_n)_{n\in \mathbb{N}}$ be a sequence of nonnegative measurable functions in $L_1[0,1]$. Assume that $$f_n \to f, \text{ a.e.}$$ and $$\int f_n h \to \int g h,\, \forall h \in C[0,1].$$

Question: Do we have $$f = g, \text{ a.e.?}$$

Remark: Of course, it is a standard result that if we allow $h$ to be any bounded measurable function, then the almost sure limit coincides with the weak limit.

$\endgroup$
2
  • 7
    $\begingroup$ This does not follow. Take $f_n$ very large on a very small set $A_n$ so that $\int f_n=1$; this set should be spread out over $(0,1)$. Then $f=0$ (definitely if $|A_n|$ is summable), $g=1$. $\endgroup$ Jan 3, 2015 at 21:07
  • $\begingroup$ Indeed, this is a nice counter-example. $\endgroup$
    – Yanqi QIU
    Jan 3, 2015 at 21:30

1 Answer 1

3
$\begingroup$

Exploiting Christian Remling's idea, we can take the following example: $$f_n:=b_n\mathbb 1\left(\bigcup_{j=1}^n\left(j /n-a_n,j/n+a_n\right) \right),$$ where the sequence $(a_n)$ is such that the series $\sum_n na_n $ converges and $na_nb_n=1$ for each $n$. Since for a continuous function $h$, the inequality $$\left|\int f_nh\mathrm dx-\frac 1n\sum_{j=1}^nh(j/n)\right| \leqslant nb_na_n \sup_{\substack{ s,t\in[0,1]\\ |s-t|\leqslant a_n } }|h(t)-h(s)|= \sup_{\substack{ s,t\in[0,1]\\ |s-t|\leqslant a_n } }|h(t)-h(s)|, $$ hence we have $$\lim_{n\to \infty}\int f_nh\mathrm dx=\int h\mathrm dx.$$

Since $\sum_n\lambda\{x\mid f_n(x)\neq 0\}\leqslant \sum_nna_n$, we have $f_n\to 0$ almost everywhere (by the Borel-Cantelli lemma).

$\endgroup$

Your Answer

By clicking “Post Your Answer”, you agree to our terms of service and acknowledge you have read our privacy policy.

Not the answer you're looking for? Browse other questions tagged or ask your own question.